A Study Of Slow Increasing Functions And Their Applications To Some Sequences Of Integers

DOI : 10.17577/IJERTV2IS60075

Download Full-Text PDF Cite this Publication

Text Only Version

A Study Of Slow Increasing Functions And Their Applications To Some Sequences Of Integers

K. Santosh Reddy*

K. Madhusudhan Reddy and B. Chandra Sekhar

Vardhaman College of Engineering, Shamshabad, Hyderabad, Andhra Pradesh, India

ABSTRACT

In this article we first define a slow increasing function. We investigate some basic properties of slow increasing function. In addition, several applications in some some sequences of integers using the theory of slow increasing functions.

KEYWORDS. Slow Increasing Functions, asymptotically equivalent, sequence of positive integers.

  1. INTRODUCTION

    Slow increasing functions are defined as follows.

    1. Definition. Let f : a, 0, be a continuously differentiable function such that

      f 0 and lim f (x) . Then f is said to be a slow increasing function (s.i.f. in short) if

      x

      lim xf (x) 0

      x f (x)

      Write F f : f

    2. Examples. (i)

      is a s.i.f..

      f (x) log x, x 1is a s.i.f.

      Note that lim f (x) lim log x and f (x) 1 ,x 1 and f is continuous

      Moreover

      x

      x

      lim

      x

      xf (x) lim 1

      x 0

      x

      (ii) f (x) loglog x, x e is also a s.i.f.

      f (x)

      x x

      log x

  2. SOME PROPERTIES

    1. Theorem. Let f , g F

      and let 0, c 0 be to constants then we have

      i) f c

      1. f c

      2. cf (iv) fg (v) f (vi) f g

      (vii) log f (viii) f g

      all lie in F .

      Proof: Given that f , g F and 0, c 0 be constants.

      Proof of (i), (ii), (iii), and (iv) follows the definition 1.1

      1. Let h f

        Note that lim h(x) lim f (x) , and h(x) f (x) 1 f (x) 0, and h is continuous

        x x

        lim xh(x) lim x f (x) 1 f (x) lim xf (x) 0.

        h f F

        Moreover

        x

        h(x)

        x

        f (x)

        x

        f (x)

        Hence

      2. Let h f g i.e h(x) f (g(x))

        Note that lim h(x) lim f (g(x)) , and h(x) f (g(x))g(x) 0, and h is continuous

        x x

        Moreover

        lim xh(x) lim xf (g(x))g(x) lim g(x) f (g(x)) xg(x) 0. Hence h f g F

        x

        h(x)

        x

        f (g(x))

        x

        f (g(x))

        g(x)

      3. Let h log f

        Note that lim h(x) lim log f (x) , and h(x) f (x) 0, and h is continuous

        x

        x

        f (x)

        Moreover

        x f (x)

        lim xh(x) lim f (x) lim x f (x) 1 0.

        Hence h log f F

        x

        h(x)

        x log f (x)

        x

        f (x) log f (x)

      4. Let h f g

      For sufficientely large x , we have 0 xf

      xf

      and 0 xg

      xg

      f g f f g g

      0 lim xh(x) lim xf lim xg 0

      By adding the above, we get

      x

      h(x)

      x f

      x g

      lim xh(x) 0

      Hence h f g F

      x h(x)

    2. Theorem. Let f , g F. Define h(x) f (x ) and k(x) f (x g(x)) for each x, then h,k F.

      Proof: Given that

      Let

      f , g F. Define h(x) f (x ) and k(x) f (x g(x)) for each x.

      h(x) f (x )

      Note that lim h(x) lim f (x ) , and h(x) f (x ) x 1 0, and h is continuous

      x

      x

      lim xh(x) lim xf (x ) x 1 lim x f (x ) 0

      Moreover

      x

      h(x)

      x

      f (x )

      x

      f (x )

      Hence

      Let k(x) f (x g(x))

      h(x) f (x ) is s.i.f.

      Note that lim k(x) lim f (x g(x)) , and

      x x

      k(x) f (x g(x)) x 1g(x) x g(x) 0 and k is continuous

      Moreover

      lim

      x

      xk(x)

      k(x)

      lim

      x

      xf (x g(x)) x 1g(x) x g(x) f (x g(x))

      lim

      x g(x) f (x g(x))

      lim

      x g(x) f (x g(x)) xg(x)

      x

      f (x g(x))

      x

      f (x g(x))

      g(x)

      0

      0

      Therefore k(x) f (x g(x)) is s.i.f. Hence h, k F

      f (x)

      d f (x) f

    3. Theorem. Let f , g F

      be such that lim

      x g(x)

      and

      dx g(x) 0. Then

      F.

      g

      Proof: Given that

      f , g F, lim

      f (x)

      and

      d f (x)

      0

      Let Moreover

      h(x)

      f (x)

      g(x)

      x g(x)

      and h(x)

      dx g(x)

      f (x)g(x) f (x)g(x)

      g(x)2

      x f (x)g(x) f (x)g(x)

      xh(x)

      g(x)2

      xf (x)

      xg(x)

      lim

      lim lim

      • lim 0

        Hence

        x

        h(x)

        x

        f (x)

        g(x)

        f F g

        x

        f (x)

        x

        g(x)

    4. Theorem. Let

      and lim h(x)

      x

      h : a, 0,

      be a continuously differentiable function such that

      h(x) 0

      1. Define g(x) h(log x) . Then g F lim h(x) 0

        x h(x)

      2. Define k(x) eh(x) .Then k F lim xh(x) 0

      x

      Proof: Given that h (x) 0 and lim h(x)

      x

      1. Define

        g(x) h(log x) then g(x) h(log x)

        x

        x h(log x)

        Suppose g F then g satisfies lim xg (x) 0

        i.e. lim x 0

        lim h (log x) 0

        x

        g(x)

        x

        h(t)

        h(log x)

        h(x)

        x h(log x)

        Put t log x so that x t lim 0

        t h(t)

        i.e.

        h(x)

        lim

        x h(x)

        0 .

        Conversely suppose

        lim 0

        x h(x)

        Put t ex

        so that

        x= log t and

        x t

        lim

        h(x)

        lim

        h(log t) 0

        tg(t)

        h(log t)

        x h(x)

        h(x)

        t h(log t)

        Now

      2. Like proof of (i)

      lim

      t

      g(t)

      lim

      t h(log t)

      lim

      x h(x)

      0.

      Hence g F

    5. Theorem. If f F

      then lim

      x

      log f (x) log x

      0.

      f (x)

      log f (x)

      f (x)

      Proof: Given that

      f F,

      lim

      lim

      (byLHospitals rule)

      x

      lim

      x

      log x

      xf '(x) f (x)

      x

      0

      1

      x

      x

      i.e. lim

      x

      log f (x) log x

      0 .

    6. Theorem. f F if and only if to each 0 there exists x such that d f (x) 0, x x

      dx x

      d f (x)

      f (x)x f (x) x 1

      f (x) xf (x)

      Proof: We have

      dx x

      x2

      x 1

      f (x)

      Suppose f F

      then lim

      x

      xf '(x) 0

      f (x)

      i.e. For each 0 there exists x such that x x

      And

      xf '(x) 0 ,

      x x

      d f (x) 0,

      x x

      f (x)

      dx x

      To prove the converse assumes that the condition holds. Let 0 be given. Then there exist x such that x x

      We have, by hypothesis

      d f (x) 0

      this implies that

      xf '(x) 0 ,

      x x

      dx x

      xf (x)

      xf '(x)

      f (x)

      i.e.

      f (x)

      0 as x lim

      x

      f (x)

      0.

      Therefore f F.

    7. Theorem. If f F then lim f (x) 0 , for all

      x x

      Proof: For any with 0 , we get by Theorem 2.6,

      d f (x) 0,

      x x

      for some x

      This implies that

      f (x)

      f (x)

      x is decreasing for x x

      dx x

      Hence bounded above, say, by M

      x

      f (x)

      That is, there exists M > 0 such that 0

      x

      M , x x

      lim f (x) lim f (x) 1 0

      x x

      x x

      x

    8. Note. We know that each f F is an icreasing function. Moreover by the above theorem it is clear

      f (x)

      that lim = 0, >0 . This shows that the increasing nature of f is slow. That is f does not increase

      x x

      rapidly. This justifies the name given to the members of F.

      From the above theorem, we have the following results.

    9. Theorem. If f F then lim f (x) 0 and lim f (x) 0.

      x x

      f (x)

      x

      Proof: In Theorem 2.7 put 1, toget lim = 0 .

      x

      If f F , then lim

      x

      x

      xf (x) 0

      f (x)

      Since

      lim f (x) =0

      we must have lim f (x)=0.

      x x

      x

    10. Theorem. Let f F then for any 1and , the series n f (n)

      n1

      diverges to .

      1

      1

      1

      Proof: We write

      n f (n)

      n1

      n f (n)

      n1 n

      we know that the series 1 diverges to

      n1 n

      Given 1 1 0

      If 0 then

      lim n 1 f (n)

      n

      n n 1

      If 0 then

      lim

      n f (n)

      lim

      n

      f (n)

      (from Theorem 2.7)

      n

      i.e. n f (n)

      n1

      diverges to

      An important byproduct of the above theorem is the following result.

      x

      t f (t) dt

    11. Theorem. Let f F. Then for any 1 and , lim a 1.

      x x 1 f (x)

      1

      Proof: From Theorem 2.10, we have lim x 1 f (x)

      n

      x

      x

      1

      lim

      x 1

      f (x) ,

      x

      1,

      From Theorem 2.10, we have t f (t) lim t f (t) dt

      x

      t f (t) dt

      t 1

      x

      a

      x f (x)

      Consider

      lim a lim

      (byLHospitalss rule)

      x x 1 f (x)

      x x 1

      x f (x)

      1

      1

      1

      f (x)

      f (x)

      x f (x)

      lim 1

      x xf (x)

      x f (x) 1

      1 f (x)

    12. Definition.Let f , g :a, 0,

      1. If lim f (x) 1, then f is is said to asymptotically equivalent to g . We describe this by writing f

        x g(x)

        • g .

      2. f

        (g) Means f

        Ag for some

        A 0. In this case we say that f is of large order g.

      3. f

      (g) Means lim f (x) 0 . In this case we say that f is of small order

      x g(x)

      g.

      f (x) xn

    13. Examples. (i) Consider

      f (x) xn , g(x) xn x, for all x 0 and

      lim

      x g(x)

      lim 1

      x xn x

      Therefore

      f g.

      1. x (10x) Because

        x 1 x 1 (10x).

        10x 10 10

      2. x 1 (x2 ) Becuase lim x 1 0.

      x x2

      As a result of the Theorem 2.11, we get the following results as particular cases.

    14. Theorem. Let

      f F. Then we have the following statements.

      x

      (i) f (t)

      a

      dt xf (x)

      (ii)

      x

      f (t)dt xf (x)

      a

      (iii)

      1 dt

      x

      x

      a f (t)

      x

      f (x)

      Proof: Let f F

      1. Put = 0 in Theorem 2.11, we get

        x

        x

        x

        f (t) dt

        lim a x

        xf (x)

        1 f (t) dt xf (x)

        a

      2. Put = 0, = 1 in Theorem 2.11, we get

        x

        f (t)dt x

        lim a 1 f (t)dt xf (x)

        x xf (x) a

      3. Put = 0, = -1 in Theorem 2.11, we get

      1

      1

      x

      dt

      x 1 x

    15. Theorem. Let f F. Then

      lim a x

      f (t) 1

      x

      f (x)

      dt

      a f (t)

      f (x)

      (i) lim f (x c) 1, For any c (ii) If f (x) is decreasing then lim f (cx) 1, for any c

      x

      f (x)

      x

      f (x)

      Proof: Let f F

      1. Case (a). Suppose c 0

        By Lagranges mean value theorem, There exists a t x, x c such that

        f x c f (x) (x c x) f (t)

        0 f x c f (x) cf (t)

        ,

        ,

        0 lim f x c f (x) lim cf (t)

        f (x)

        t x, x c

        f (x)

        x

        f (x)

        x

        f (x)

        lim f x c 1 0 , since lim f (x) 0 (by Theorem 2.9)

        x

        f (x)

        x

        lim f x c 1.

        Case (b). Suppose c 0

        x f (x)

        By Lagranges mean value theorem there exists t x c, x such that

        f (x) f x c (x x c) f (t)

        0 f (x) f x c cf (t)

        f (x)

        ,

        ,

        0 lim f (x) f x c c lim f (t)

        f (x)

        t x c, x

        x

        f (x)

        x f (x)

        lim f x c 1 0 , since lim f (x) 0

        (by Theorem 2.9)

        x

        f (x)

        x

        lim f x c 1.

      2. Case (a). Suppose c 1

      x f (x)

      By Lagranges mean value theorem there exists t x, cx such that

      f cx f (x) (cx x) f (t)

      0 f cx f (x) (c 1)xf (t)

      f (x)

      ,

      ,

      0 lim f cx f (x) (c 1) lim xf (t)

      f (x)

      t x, cx

      And

      f (x) is decreasing

      x

      f (x)

      f (x) f (t)

      x

      f (x)

      There fore

      lim f cx 1 0 , since lim f (x) 0

      (by Theorem 2.9)

      x

      f (x)

      x

      lim f cx 1.

      Case (b). Suppose c 1

      x f (x)

      By Lagrange mean value theorem there exists t cx, x such that

      f (x) f cx (x cx) f (t)

      0 f (x) f cx (1 c)xf (t)

      f (x)

      ,

      ,

      0 lim f (x) f cx (1 c) lim xf (t)

      f (x)

      t cx, x

      x

      f (x)

      x

      f (x)

      And

      f (x) is decreasing

      f (x) f (t)

      There fore

      lim f cx 1 0 , since lim f (x) 0

      (by Theorem 2.9)

      x

      f (x)

      x

      lim f cx 1.

      x f (x)

    16. Theorem. Suppose f F is such that f (x) is decreasing. If 0 c1 c2 and g is a function such that

      f (g(x)x)

      1 2

      1 2

      c g(x) c then lim

      x

      Proof: Suppose f F is such that f (x) is decreasing

      f (x)

      1.

      If 0 c1 g(x) c2 f (c1x) f (g(x)x) f (c2 x) since f is decreasing

      f (c1x)

      f (g(x)x)

      f (c2 x)

      lim f (c1x) lim f (g(x)x) lim f (c2 x)

      f (x)

      f (x)

      f (x)

      x

      f (x)

      x

      f (x)

      x

      f (x)

      1 lim

      x

      f (g(x)x) 1

      f (x)

      (By Theorem 2.15)

      lim

      x

      f (g(x)x)

      f (x)

      1.

  3. APPLICATIONS OF SLOW INCREASING FUNCTIONS TO SOME SEQUENCES OF INTEGERS

This topic is aimed at applications in some special sequences of positive integers. Infact several asymptotic results related to these integer sequences are derived by using the theory of Slow Increasing Functions.

We begin with the following important definition.

Let f F . Through out this chapter (an ) denotes astrictly increasing sequence of positive integers such that

a 1 And lim an

1 for some s 1.

(1)

1 n ns f (n)

n

n

There exist several such sequences.

i.e.

a ns f (n)

For example an pn , the sequence of prime numbers in increasing order,

f (x) log x and s 1.

By prime number theorem we have lim pn 1.

n n log n

    1. Definition. Let (an ) be asequence as described above. Then for any x 0 , define (x) 1

      an x

      The number of an that do not exceed x .

    2. Theorem. If (an ) satisfies (1) and g F, then

      1. a a

      2. lim an1 an =0

      3. l o g a log a

      4. n1 n

        n an

        n1 n

        g(an1) g(an )

      5. log an s log n (vi) loglog an loglog n

      (x)

      (vii) lim =0

      Proof: Let (an ) satisfies (1) and g F

      x x

      a (n 1)s f

      (n 1)

      1 s

      f (n 1)

      1. Consider

        lim

        n1

        = lim

        li m 1

        lim =1 By

        Theorem 2.15

        n an

        n

        ns f (n)

        n

        n n

        f (n)

      2. We have

        a a

        an1 an

        lim an1 1 lim an1 1 0 lim an1 an 0

        n1 n

        n an

        n an

        n an

      3. Consider

        lim an1 1 log lim an1 log1 lim log an1 0

        n an

        n an

        n

        an

        limlog an1

        • log an

        0 lim log an1 log an 0

        n

        n

        log an

        lim log an1 1

        i.e. l o g an1

        • log an

          n

          log an

      4. As a a , g F, we have

        lim g(an1 ) 1

        g(a ) g(a )

        n1 n

        n g(an )

        n1 n

      5. We have

        an

        ns f (n)

        log an

        log ns f (n)

        log an

        slog n log f (n)

        log an

        1 log f (n)

        lim log an = 1 1 lim log f (n)

        lim log an 1 By

        slog n

        Theorem 2.5

        slog n

        n slog n s n

        log n

        n s log n

      6. We have logan s log n

        i.e. logan s log n

        loglogan logs log n

        loglogan log s loglog n

        loglogan

        log s 1

        lim loglogan log s lim 1 1

        log log n

        log log n

        n log log n

        n log log n

        lim loglogan 1

        n log log n

        i.e. loglogan

        • loglog n.

      7. We have

      (x) 1

      an x

      Let n be the largest index such that a

      x then

      (x) n

      (x) n0 n0

      0 n0

      0 x x a

      n

      n

      0

      0 lim (x) lim n0

      0 lim (x) 0

      lim (x) 0

      x x

      n0 a

      n

      n

      0

      x x

      x x

    3. Theorem. Suppose an

      g(a ) lg(n) g( (x)) 1 g(x).

      satisfies (1) and g F,

      then for l 1,

      n

      Proof: First suppose that

      l

      g( (x)) 1 g(x) g( (a )) 1 g(a )

      l n l n

      g(n) 1 g(a ) g(a ) lg(n) . ( (a ) n )

      l n n n

      Conversely suppose

      g(a ) lg(n) g(a ) lg( (a ))

      lim g( (an )) 1

      n n n

      n 1 g(a )

      l n

      (2)

      If an x an1 we have

      (an ) x an1

      g( (an )) g( (x)) g( (an1))

      since

      And

      g F.

      g(a ) g x g a

      1 g(a ) 1 g x 1 g a

      (l 1)

      n n1

      l n l l n1

      lim g( (an )) lim g( (x)) lim g( (an ))

      n

      1 g(a )

      x

      1 g(x)

      n

      1 g(a )

      l n l l n

      (a

      a )

      1 lim g( (x)) 1 by 2

      n1 n

      x

      1 g x l

      lim g( (x)) 1

      g( (x)) 1 g(x).

      x

      1 g x l

      l

    4. Theorem. Suppose an

      satisfies (1) and g F , then (i)

      log an

      • s log n log (x) 1 log x

        s

        s

        (ii) loglog an log logn log log (x) log log(x).

        Proof: Given an satisfies (1) and g F

        1. In Theorem 3.3 put g(an ) log an , g(n) log n and l s . And we have logan s log n.

        2. In Theorem 3.3 put g(an ) loglog an , g(n) loglog n and l s

        And we have loglogan loglog n.

        bn

        bn

        We make use the following well known result in proving theorems to follow.

    5. Result. Let

      bn

      and

      cn be two series of positive terms such that lim

      1. If

      cn is divergent

      n1

      n1

      n cn

      n1

      n

      n

      bk

      n

      n

      then it is known that lim k 1 1.

      n

      ck

      k 1

    6. Theorem. Let f F, an satisfies (1) and f (an )

      f (n). Then

      k

      k

      x x 1

      an nf (n) (x)

      (x) dt f (a ) x.

      f (x) f (t)

      Proof: Given an

      (3)

      satisfies (1) and f F

      a

      and

      ak x

      f (an )

      f (n)

      First suppose that

      (x) x

      f (x)

      (an )

      an

      f (an )

      n an

      f (an )

      an nf (an )

      an nf (n)

      By (3)

      Conversely suppose

      a nf (n)

      a (a ) f (n)

      (a ) an

      n

      (a )

      n n

      an by (3)

      n

      lim (an ) 1

      f (n)

      n

      (4)

      f (an )

      n an

      f (an )

      If an x an1 we have

      (an ) x an1

      We have by Theorem 2.6

      f (x) x

      has negative derivative

      f (x) x

      is decreasing

      x

      f (x)

      is increasing

      an

      x

      an1

      lim (an ) lim (x) lim (an )

      (a a )

      f (a )

      f (x)

      f (a )

      n an x x n an

      n1 n

      n n1

      f (an )

      f (x)

      f (an )

      1 lim x 1

      By (4)

      x

      lim 1

      (x) x

      x x

      f (x)

      x x

      f (x)

      f (x)

      Suppose

      (5)

      (x)

      x 1 x

      x

      f (x)

      Then we have from Theorem 2.14

      (6)

      dt

      a f (t)

      x 1

      f (x)

      Hence from (5) and (6)

      (x) f t dt.

      a ( )

      Also we have from Theorem 2.14

      x

      f (t)dt xf (x)

      a

      n n

      And since f (x) is increasing, we get f (k) f (x)dx h(n) nf (n)

      k 1 a

      (7)

      Given that

      f (an )

      f (n)

      n n

      n n

      f (ak ) f (k)

      By Result 3.5

      n

      n

      f (ak ) nf (n)

      By (7)

      k 1

      f (a ) nf (a )

      k 1

      f (a ) (a ) f (a )

      k 1

      lim

      (an ) 1

      k n

      ak an

      ak an

      k n n

      n

      f (ak )

      (8)

      If

      an x an1 we have

      (an ) x an1

      ak an

      f (an )

      And

      f (an ) f x f an1

      f (ak ) f (ak )

      f (ak )

      ak an ak x ak an1

      lim

      (an )

      lim x lim

      an

      (a

      a )

      n

      f (a )

      x

      f (a )

      n

      f (a )

      n1 n

      k k k

      k k k

      ak an ak x ak an

      f (an )

      x

      f x

      f an

      x

      f x

      f x

      f (ak )

      1 lim

      x

      1

      f (ak )

      By (8)

      lim

      x

      1

      f (ak )

      x ak x

      ak x

      f x

      f (ak )

      ak x

      f x

      x

      f (x)

      a x

      k

      k

      f x

      ak x

      f (ak ) x

    7. Theorem. Let f F,

      an satisfies (1) and

      f (an ) lf (n). Then

      1 1 1

      l

      l

      x

      11

      1 1 1

      n

      n

      a ns f (n) (x)

      l s xs

      1

      s

      (x)

      s

      t

      t

      s

      s

      1 dt f (ak )s l s xs .

      f (x)s

      a f (t)s

      ak x

      Proof: Given an satisfies (1) and f F

      (9)

      and

      f (an ) lf (n)

      Suppose

      (x)

      1 1

      l s xs

      1

      f (x)s

      (an )

      1 1

      l s an s

      1

      f (an )s

      1 1

      l s a s

      n n

      n n

      1

      f (an )s

      (9)

      ns

      lan

      f (an )

      an

      • ns f (n) By

        Conversely suppose

        a ns f (n)

        1 1

        an s nf (n)s

        lim

        n

        n

        n

        (an ) 1

        1 1

        l sans

        By (9)

        1

        f (an )s

        (10)

        If an x an1 we have

        (an ) x an1

        We have by Theorem 2.6

        increasing

        f (x) x

        has negative derivative

        f (x) x

        is decreasing

        x is

        f (x)

        an

        x

        an1

        lim (an )

        lim x

        lim an

        (a a )

        f (a ) f (x) f (a )

        n 1 1

        x 1 1

        n 1 1

        n1 n

        n n1

        l s a s

        l s xs

        l s a s

        n n

        1

        f (an )s

        1

        f (x)s

        1

        f (an )s

        1 lim x 1

        x

        lim 1

        (x)

        1 1

        l s xs

        x

        1 1

        l s xs

        1

        By (10)

        x

        1 1

        l s xs

        1

        1

        f (x)s

        f (x)s f (x)s

        s

        s

        an n f (n) (x)

        1 1

        l s xs

        1

        f (x)s

        (11)

        x

        t f (t) dt

        x x 1 f (x)

        We have from Theorem 2.11

        lim a 1

        1

        1

        t f (t) dt

        x x f (x)

        1

        a 1

        x 1 1

        1 1 1 1

        In above equation put 1 1

        and 1

        1

        t s f (t) s dt

        s f (x) s

        s s a

        1 1 1

        s

        x

        x

        x 1 1 1

        1 x 1 1 1 1

        1 t s

        s xs

        1 dt 1

        l s t s

        s

        1 dt

        l s xs

        1

        a

        (12)

        f (t)s

        f (x)s

        1

        l s x

        1 1

        t s

        a f (t)s

        f (x)s

        From (11) and (12)

        (x)

        1 dt

        s

        s

        a f (t)s

        Also we have from Theorem 2.14

        n

        x

        f (t)dt xf (x)

        a

        n

        and

        f (x) is increasing

        Now

        f (k) f (x)dx h(n) nf (n)

        k 1 a

        (13)

        Given that

        f (a)n lf (n)

        n n

        n n

        f (ak ) l f (k)

        By Result

        k 1

        3.5

        k 1

        n

        n

        f (ak ) lnf (n)

        k 1

        By (13)

        ak an

        f (ak ) lnf (an )

        ak an

        f (ak ) l (n) f (an )

        f (ak )

        (a )

        (a ) ak an

        n

        n

        lim n 1

        (14)

        lf (an )

        n

        f (ak )

        If an x an1 we have

        ak an

        lf (an )

        (an ) x an1

        And

        f (an ) f x f an1

        lf (an ) lf x lf an1

        f (ak ) f (ak )

        f (ak )

        ak an ak x ak an1

        lim

        (an )

        lim x lim

        an

        (a

        a )

        n

        f (a )

        x

        f (a )

        n

        f (a )

        n1 n

        k k k

        k k k

        ak an ak x ak an

        lf (an )

        lf x

        lf an

        1 lim

        x 1

        By (14)

        lim

        x 1

        x

        f (ak )

        x

        f (ak )

        ak x

        lf x

        x

        x

        f (ak )

        ak x

        lf x

        k

        k

        f (ak )

        ak x

        lf x

        1 1 1

        x a x

        f (a ) lx

        f (a )s

        l s xs .

        f (x) lf x

        k

        ak x

        k

        ak x

    8. Theorem. If

      g(x) is a f.s.i. and

      g(a ) lg(n) where a

      satisfies (1), then

      n n

      k

      k

      g(a )

      (x) ak x , for all real .

      g(x)

      Proof: Given that g(x) is a f.s.i. and

      g(a ) lg(n) where a

      satisfies (1)

      We have from Theorem 2.14

      n

      n

      n

      n

      x

      g(t) dt xg(x)

      a

      n n

      And since g(x) is increasing, we get

      g(k) g(x) dx h(n) ng(n)

      (15)

      Given that (16)

      k 1

      g(an ) lg(n)

      a

      n

      n

      g(a ) l g(n)

      g(a ) l g(n)

      g(a ) l g(n)

      n n

      k

      By Result 3.5

      k 1 k 1

      g(a ) l ng(n)

      g(a ) l ng(n)

      n

      k

      k 1

      By (15)

      g(a ) ng(a )

      k n

      k n

      ak an

      g(a )

      By (16)

      g(a ) (a )g(a )

      k

      n

      n

      (a ) ak an

      lim (an ) 1

      ak an

      k n n

      n g(a )

      n

      g(a )

      k

      k

      ak an

      n

      n

      g(a )

      (17)

      If an x an1 and 0 (an ) (x) (an1 )

      We have g F

      g(a ) g(x) g(a ) g(a ) g(x) g(a )

      n n1 n n1

      g(a )

      g(a )

      g(a )

      ak an

      k k k ak x ak an1

      k k k

      k k k

      g(a )

      g(a )

      g(a )

      ak an

      ak x

      ak an1

      n

      n

      g(an1 )

      g(x)

      g(a )

      lim

      (an )

      lim (x) lim

      (an )

      (a

      a )

      n

      g(a )

      x g(a )

      n

      g(a )

      n1 n

      k k k

      ak an ak x ak an

      g(a ) g(x) g(a )

      n n

      1 lim (x) 1

      x g(a )

      k

      k

      By (17)

      lim (x) 1

      x g(a )

      k

      k

      ak x

      g(x)

      ak x

      g(x)

      g(a )

      k

      k

      (x)

      ak x

      g(x) .

      If an x an1 and 0

      (an ) (x) (an1 )

      We have g F

      g a g( x ) g a( )

      g a ( g x) g a( )

      ( 0)

      n n1

      n1 n

      lim

      (an )

      lim (x) lim

      (an )

      (a a )

      n

      g(a )

      x g(a )

      n

      g(a )

      n n1

      k k k

      ak an ak x ak an

      g(a ) g(x) g(a )

      n n

      1 lim (x) 1

      x g(a )

      k

      k

      By (17)

      lim (x) 1

      x g(a )

      k

      k

      ak x

      g(x)

      ak x

      g(x)

      k

      k

      g(a )

      (x) ak x

      Hence

      g(x) .

      k

      k

      g(a )

      (x) ak x

      g(x)

      For all

      real.

    9. Theorem. If an satisfies (1) and (x) 1, the number of primes up to x, then

      pn x

      (x) (x).

      a

      a

      k

      k

      Proof: The ak x are a1, a2,…, a ( x) .

      Let us write

      k x, (k 1, 2,…, (x))

      log ak

      log x

      k log ak

      log x

      k

      k

      k

      log x

      log ak

      (k 1, 2,…, (x))

      We know that

      (x) (x)

      ( x)

      k

      ( x)

      k

      ( x) 1

      log x log a

      k 1

      (18)

      k 1

      1 1

      k 1 k

      From theorem 3.2, We have logan s log n

      log an s log n

      ( x) 1 1

      1 ( x) 1

      log a

      • log a s log k

        By Result 3.5 (19)

        k 1 k

        1 k 2

        x 1 x

        2

        2

        x 1 x

        We have from Theorem 2.14

        (20)

        dt

        a f (t)

        f (x)

        log tdt log x

        1 1 ( x) 1

        ( x) 1

        (x)

        Now

        (20)

        log a1

        s k 2 log k

        dt (1) By

        2 log t s log x

        ( x) 1

        (x)

        From Equation (19) and above equation

        log a

      • s log x

        k 1

        k

        h(x) (x) log x

        From Equation (18) and above equation

        (x) (x)

        s log x

        (h(x) 1)

        1 (x) h(x) log x 1 lim (x) lim h(x) log x

        (x)

        (21)

        s log x

        x (x)

        x

        s log x

        We have from Theorem 3.4 of (i)

        log (x) 1 log x

        lim

        log x 1

        s

        (x)

        x s log (x)

        (x)

        x

        x

        x

        x

        Using this inEquation (21), we get

        1 lim (x) 1

        (h(x) 1)

        lim (x) 1

        Hence

    10. Theorem. Let f F, an satisfies (1).

(x) (x).

n ns 1 f (n) n

Then (i)

ak

k 1

s 1

  • s 1 an

    ( 0)

    (ii)

    a (x) x

    ( 0)

    n

    an x

    s 1

    Proof: Given that Let f F, an satisfies (1)

    n

    n

    1. Let us consider the sum 1 2 … (n 1) (ks f (k)s )

      k n

      (22)

      n

      n

      n

      n

      Where n is positive integer in the interval a,

      From Equation (1) we have (23)

      a ns f (n)

      a ns f (n) ( 0)

      n n

      We know that xs f (x)s is increasing

      (ks f (k)s ) xs f (x) dx (ns f (n)

      (24)

      x

      k n n

      t f (t) dt

      x x 1 f (x)

      From Theorem 2.13, we have

      lim

      x

      a

      x 1 f (x)

      1

      t f (t)

      a

      dt

      1

      1

      Put s

      (25)

      and

      in above equation, we get

      n

      xs f (x) dx

      n

      ns 1 f (n)

      s 1

      From Eqations (22), (24) and (25), we get

      n

      n

      1 2 … (n 1)

      k n

      s 1

      n f (n)

      n f (n)

      (ks f (k)s )

      s 1

      n nf (n)

      s

      s

      s 1

      ) (26)

      s s

      na

      n

      n

      n

      n

      1 2 … (n

      1) (k f (k) )

      k n

      s 1

      By (23) (27)

      FromEquations (23), (26) and (27) and using Result 3.5, we get

      n ns 1 f (n) n

      ak

      k 1

      s 1

  • s 1 an

( 0)

  1. If

an x an1 and 0

(an ) x an1

And

a x a

(s +1) a (s +1) a (s +1) a

n n1

k k k ak an ak x ak an1

lim

(an )

lim (x) lim

(an )

( a a )

n (s +1)

a

x (s +1) a

n

(s +1) a

n n1

k k k

ak an ak x ak an

a x a

n n

(28)

k

k

n na

(n)a

s 1 a

a

a

We have

a n

a n

(a )

ak an

k

k 1

s 1

k

ak an

s 1 n

(x)

lim

n

(an )

k

k

n

n

s 1 a =1

ak an

a

a

n

x

x

(x)

Equation (28) implies

1 lim (s +1) a 1

k

k

lim (s +1) a 1

k

k

x

x

ak x

x

ak x

x

k

k

(s +1) a

(x)

(x) ak x

x

a

n

n

an x

s 1

x . ( 0).

We apply the results discussed in this article to look into some of the applications in number theory.

Acknowledgments. The author would like to thank the anonymous referees for a careful reading and evaluating the original version of this manuscript.

REFERENCES

1

2

3

Appl.

4

G. H. Hardy and E. M. Wright, An Introduction to the Theory of Numbers, Fourth Edition, 1960.

R. Jakimczuk. Functions of slow increase and integer sequences, Journal of Integer Sequences, Vol. 13 (2010), Article 10.1.1.

R. Jakimczuk. A note on sums of powers which have a fixed number of prime factors, J.Inequal. Pure

Math. 6 (2005), 5-10.

R. Jakimczuk. The ratio between the average factor in a product and the last factor, mathematical

sciences:

Quarterly Journal 1 (2007), 53-62.

5 Y. Shang, On a limit for the product of powers of primes, Sci. Magna 7 (2011) 31-33.

6 J. Rey Pastor, P. Pi Calleja and C. Trejo, An alisis Matem arico, Volumen I, Octava Edicion, Editorial Kapelusz,

1969.

Leave a Reply